Skip to main content
Log in

An Averaging Principle for Stochastic Flows and Convergence of Non-Symmetric Dirichlet Forms

  • Published:
Potential Analysis Aims and scope Submit manuscript

Abstract

We study diffusion processes and stochastic flows which are time-changed random perturbations of a deterministic flow on a manifold. Using non-symmetric Dirichlet forms and their convergence in a sense close to the Mosco-convergence, we prove that, as the deterministic flow is accelerated, the diffusion process converges in law to a diffusion defined on a different space. This averaging principle also holds at the level of the flows. Our contributions in this article include:

  • a proof of an original averaging principle for stochastic flows of kernels;

  • the definition and study of a convergence of sequences of non-symmetric bilinear forms defined on different spaces;

  • the study of weighted Sobolev spaces on metric graphs or “books”.

This is a preview of subscription content, log in via an institution to check access.

Access this article

Price excludes VAT (USA)
Tax calculation will be finalised during checkout.

Instant access to the full article PDF.

Institutional subscriptions

Similar content being viewed by others

References

  1. Bakry, D., Gentil, I., Ledoux, M.: Analysis and Geometry of Markov Diffusion Operators. Springer (2014)

  2. Barret, F., von Renesse, M.: Averaging principle for diffusion processes via Dirichlet forms. Potential Anal. 41(4), 1033–1063 (2014)

    Article  MathSciNet  Google Scholar 

  3. Billingsley, P.: Convergence of Probability Measures. Wiley, New York (1968)

    MATH  Google Scholar 

  4. Bishop, E., de Leeuw, K.: The representations of linear functionals by measures on sets of extreme points. Ann. Inst. Fourier. Grenoble 9, 305–331 (1959)

    Article  MathSciNet  Google Scholar 

  5. Breiman, L.: Probability, Classics in Applied Mathematics. SIAM (1992)

  6. Brezis, H.: Analyse fonctionnelle, Collection Mathematiqueś Appliquées pour la Maîtrise. [Collection of Applied Mathematics for the Master’s Degree]. Masson, Paris (1983)

    Google Scholar 

  7. Cerrai, S., Freidlin, M: Fast flow asymptotics for stochastic incompressible viscous fluids in \(\mathbb {R}^{2}\) and SPDEs on graphs. Probab. Theory Related Fields 173, 491–535 (2019)

    Article  MathSciNet  Google Scholar 

  8. Ethier, S.N., Kurtz, T.G.: Markov Processes. Wiley, New York (1986)

    Book  Google Scholar 

  9. Freidlin, M.I., Wentzell, A.D.: Random Perturbations of Dynamical Systems. Springer (2012)

  10. Funaki, T., Nagai, H.: Degenerative convergence of diffusion process toward a submanifold by strong drift. Stochastics Stochastics Rep. 44(1–2), 1–25 (1993)

    MathSciNet  MATH  Google Scholar 

  11. Hajłasz, P: Sobolev spaces on an arbitrary metric space. Potential Anal. 5, 403–415 (1996)

    MathSciNet  MATH  Google Scholar 

  12. Hajri, H., Raimond, O.: Stochastic flows and an interface SDE on metric graphs. Stochastic Process Appl. 126(1), 33–65 (2016)

    Article  MathSciNet  Google Scholar 

  13. Hasselblatt, B., Katok, A.: Principal Structures, Handbook of Dynamical Systems, vol. 1A, 1–203. North-Holland (2002)

  14. Hasselblatt, B., Katok, A.: Introduction to the Modern Theory of Dynamical Systems. Cambridge University Press (1995)

  15. Hino, M.: Convergence of non-symmetric forms. J. Math. Kyoto Univ. 38(2), 329–341 (1998)

    MathSciNet  MATH  Google Scholar 

  16. Kuwae, K., Shioya, T.: Convergence of spectral structures: A functional analytic theory and its applications to spectral geometry. Comm. Anal. Geom. 11 (4), 599–673 (2003)

    Article  MathSciNet  Google Scholar 

  17. Le Jan, Y., Raimond, O.: Flows, coalescence and noise. Ann. Probab. 32(2), 1247–1315 (2004)

    MathSciNet  MATH  Google Scholar 

  18. Le Jan, Y., Raimond, O.: Integration of Brownian vector fields. Ann. Probab. 30(2), 826–873 (2002)

    MathSciNet  MATH  Google Scholar 

  19. Ma, Z.M., Röckner, M.: Introduction to the Theory of (Nonsymmetric) Dirichlet Forms. Springer (1992)

  20. Mattingly, J.C., Pardoux, E.: Invariant measure selection by noise. An example. Discrete Contin. Dyn. Syst. 34(10), 4223–4257 (2014)

    Article  MathSciNet  Google Scholar 

  21. Neveu, J.: Bases mathématiques du calcul des probabilités, Deuxième édition, revue et corrigée. Masson et Cie, Éditeurs, Paris (1970)

    Google Scholar 

  22. Oshima, Y: Semi-Dirichlet Forms and Markov Processes. Walter de Gruyter & Co., Berlin (2013)

    Book  Google Scholar 

  23. Phelps, R.R.: Lectures on Choquet’s Theorem. Lecture Notes in Mathematics, vol. 1757. Springer, Berlin (2001)

    Google Scholar 

  24. Raugi, A: A probabilistic ergodic decomposition result. Ann. Inst. Henri Poincaré Probab. Stat. 45(4), 932–942 (2009)

    Article  MathSciNet  Google Scholar 

Download references

Acknowledgments

The authors are part of the LABEX MME-DII. This research has been conducted within the FP2M federation (CNRS FR 2036).

Author information

Authors and Affiliations

Authors

Corresponding author

Correspondence to Florent Barret.

Additional information

Publisher’s Note

Springer Nature remains neutral with regard to jurisdictional claims in published maps and institutional affiliations.

Appendix:

Appendix:

Notation: In a topological space \(A\Subset B\) means that \(\overline {A}\subset \mathring {B}\) and \(\overline {A}\) is compact.

1.1 Weighted Sobolev Spaces

For k ≥ 1, the class of Muckenhoupt weights A2 consists all mappings \(\omega :\mathbb {R}^{k}\to [0,\infty ]\), for which there is a constant C such that for all ball B in \(\mathbb {R}^{k}\), we have

$$\frac{1}{|B|}\left( {\int}_{B} \omega(x) dx\right)\times \frac{1}{|B|}\left( {\int}_{B} \frac{1}{\omega(x)} dx\right) \le C.$$

For m ≥ 2, set \(B_{m,1}:=\left (\cup _{i=1}^{m} ]0,\infty [\times \{i\}\right )\cup \{0\}\), equipped with the distance d1 defined by d1((x,i), (y,i)) = |xy| and if ij, d1((x,i), (y,j)) = x + y, and d1((x,i), 0) = x. Let \(i_{1}:B_{m,1}\to \{0,\dots ,m\}\) be defined i1(x,i) = i and i1(0) = 0. Then Bm,1 is a metric graph constituted of m half lines joined at 0. Set also \(B_{m,2}:=\mathbb {R}\times B_{m,1}\), equipped with the distance d2 defined by d2((x,i), (y,i)) = ∥xy∥, \(d_{2}((x,i),(y,j))=\|x-y^{\prime }\|\) if ij and where \(y^{\prime }=(y_{1},-y_{2})\), and d2((x,i),y) = ∥xy∥ if y = (y1, 0), where ∥⋅∥ is the Euclidean norm on \(\mathbb {R}^{2}\). Then Bm,2 is a metric space constituted of m half planes joined along a line. Let also \(i_{2}: B_{m,2}\to \{0,\dots ,m\}\) be defined by i2(x1,x2,i) = i and i2(x1, 0) = 0. To simplify the notation we will simply denote d1 and d2 by d. For 1 ≤ im, we will use the notation (0,i) = 0 ∈ Bm,1 and for \(x_{1}\in \mathbb {R}\), (x1, 0,i) = (x1, 0) ∈ Bm,2.

For m = 1 and k ∈{1, 2}, set \(B_{1,k}=\mathbb {R}^{k}\).

Let \(\omega : B_{m,k}\to [0,\infty ]\) be such that \(\omega \in L^{1}_{loc}(B_{m,k})\), i.e. such that for all \(A\Subset {\Omega }\), \(\mu (A):= {\int \limits }_{A} \omega (x) dx < \infty \), with dx the measure on Bm,k that coincides with the Lebesgue measure on Ei := {xBm,k : ik(x) = i} for each i. For ij, set Ei,j := EiEjE0 (which is isometric to \(\mathbb {R}^{k}\), and will be thus identified to \(\mathbb {R}^{k}\)).

In the following, we fix k ∈{1, 2} and m ≥ 1 and we let Ω be an open subset of Bm,k. For i ∈{1,⋯ ,m}, set Ωi = Ω ∩ Ei and for 1 ≤ ijm, set Ωi,j = Ω ∩ Ei,j. Then Ωi and Ωi,j are open subsets of \(\mathbb {R}^{k}\). Denote by L2(Ω,ω) the space of all measurable functions f on Bm,k such that \({\int \limits }_{\Omega } f^{2}(x) \omega (x) dx < \infty \). For a function fL2(Ω,ω), weakly differentiable on Ωi,j for all 1 ≤ ijm, define the norm of f by

$$ \|f\|^{2}_{W^{1}({\Omega},\omega)}={\int}_{\Omega} \big(f^{2} + \|\nabla f\|^{2}\big)(x) \omega(x) dx. $$
(A.1)

Let W1(Ω,ω) be the completion with respect to the norm \(\|\cdot \|_{W^{1}({\Omega },\omega )}\) of the vector space of the functions fL2(Ω,ω), that are weakly differentiable on Ωi,j for all 1 ≤ ijm and with \(\|f\|_{W^{1}({\Omega },\omega )}<\infty \). Suppose also that \(\frac {1}{\omega }\in L^{1}_{loc}({\Omega })\). Then, for all \(i\in \{1,\dots ,k\}\) (resp. all 1 ≤ ijk), the restriction of fW1(Ω,ω) to Ωi (resp. to Ωi,j) belongs to \(W^{1,1}_{loc}({\Omega }_{i})\) (resp. to \(W^{1,1}_{loc}({\Omega }_{i,j})\).

We also define H1(Ω,ω) (resp. \({H^{1}_{0}}({\Omega },\omega )\)) to be the completion of C(Ω) ∩ W1(Ω,ω) (resp. of Cc(Ω) ∩ W1(Ω,ω)), with respect to \(\|\cdot \|_{W^{1}({\Omega },\omega )}\). Define also \({W^{1}_{0}}({\Omega },\omega )\) to be the set of all fW1(Ω,ω) such that the function F = f1ΩW1(Bm,k,ω). Equipped with the inner product

$$ \langle f,g\rangle_{W^{1}({\Omega},\omega)}={\int}_{\Omega} (fg+\nabla f\cdot\nabla g)(x)\omega(x)dx, $$
(A.2)

W1(Ω,ω), \({W^{1}_{0}}({\Omega },\omega )\), H1(Ω,ω) and \({H^{1}_{0}}({\Omega },\omega )\) are Hilbert spaces.

Lemma A.1

Let m ≥ 1 and \(O\Subset {\Omega }\) be open subsets of Bm,k. Then there is δ > 0 such that Kδ := {xBm,k : d(x,O) ≤ δ} is a compact set with OKδ ⊂Ω. Suppose that there is \(\bar {\omega }:B_{m,k}\to [0,\infty ]\) such that \(\bar {\omega }=\omega \) on Ω and such that

  • when m ≥ 2, for all ij, the restriction \(\bar {\omega }_{i,j}\) of \(\bar {\omega }\) to Ei,j belongs to the class A2.

  • when m = 1, \(\bar {\omega }\) belongs to the class A2.

Then if \(f\in {W^{1}_{0}}(O,\omega )\), there is gL2(O,ω) such that g = 0 on OKδ and such that for all (x,y) ∈ O2,

$$|f(x)-f(y)|\le d(x,y)\big(g(x)+g(y) \big).$$

Moreover, there is a sequence of lipschitzian functions \(f_{n}\in {W^{1}_{0}}(O,\omega )\) such that \(\lim _{n\to \infty }\|f-f_{n}\|_{W^{1}(O,\omega )}=0\).

Proof

We only consider the case m ≥ 2, the case m = 1 being simpler.

The existence of δ and Kδ is a standard exercise.

Let \(f\in {W^{1}_{0}}(O,\omega )\) and denote by fi,j the restriction f to Ei,j, and set Oi,j := OEi,j. Then \(f_{i,j}\in {W^{1}_{0}}(O_{i,j},\bar {\omega }_{i,j})\). Set F := f1O and Fi,j the restriction of F on Ei,j. Then \(F_{i,j}\in W^{1}(E_{i,j},\bar {\omega }_{i,j})\). Recall that Ei,j is isometric to \(\mathbb {R}^{k}\). Note that for xOi = OEi, we have for all ji, f(x) = fi,j(x) = Fi,j(x).

Since \(\bar {\omega }\in A_{2}\), we have (see [11]) \(G_{i,j}\in L^{2}(E_{i,j},\bar {\omega }_{i,j})\) such that for all \((x,y)\in E_{i,j}^{2}\),

$$|F_{i,j}(x)-F_{i,j}(y)|\le d(x,y)\big(G_{i,j}(x)+G_{i,j}(y)\big).$$

For x ∈Ω, define \(g(x):={\sum }_{j\ne i} G_{i,j}(x)\) if x ∈Ωi. Then gL2(Ω,ω), and we have for all (x,y) ∈Ω2,

$$|f(x)-f(y)|\le d(x,y)\big(g(x)+g(y)\big).$$

Set now \(g_{\delta }:=g 1_{K_{\delta }}+\delta ^{-1}|f|\). Then, we have that for all (x,y) ∈Ω,

$$|f(x)-f(y)|\le d(x,y)\big(g_{\delta}(x)+g_{\delta}(y)\big).$$

Indeed, this inequality is straightforward to check if \((x,y)\in K_{\delta }^{2}\) or if (x,y) ∈ (Ω ∖ O)2. If (x,y) ∈ O × (Ω ∖ Kδ), we have d(x,y)(gδ(x) + gδ(y)) ≥ d(x,y)δ− 1|f(x)|≥|f(x) − f(y)|. This shows the first part of the lemma.

For the second part, it suffices to follow the proof of Theorem 5 of [11] (with the function gδ, and since f = fλ on Eλ and that \(E_{\lambda }\supset {\Omega }\setminus K_{\delta }\), we have fλ = 0 sur Ω ∖ Kδ). □

For an open set U ⊂Ω, define the capacity of U by

$$\text{Cap}(U):= \inf\{\|h\|_{W^{1}({\Omega},\omega)}^{2}: h\ge 1 \text{ on} U\text{ and } h\in W^{1}({\Omega},\omega)\}.$$

Lemma A.2

Let m ≥ 1, Ω an open subset of Bm,k and \(\omega :{\Omega }\to [0,\infty ]\) a measurable mapping. For R > 0, set ΩR := {x ∈Ω : ∥x∥ < R}. Suppose that, for all R > 0, there exists a non decreasing sequence of open subsets (ΩR,n)n≥ 1 such that

  1. (i)

    n≥ 1ΩR,n = ΩR,

  2. (ii)

    \(\lim _{n\to \infty } \text {Cap}({\Omega }_{R}\setminus \overline {\Omega }_{R,n})=0\),

  3. (iii)

    for all n ≥ 1, \({W^{1}_{0}}({\Omega }_{R,n},\omega )={H^{1}_{0}}({\Omega }_{R,n},\omega )\).

Then we have \(W^{1}({\Omega },\omega )={H^{1}_{0}}({\Omega },\omega )\).

Proof

Let fW1(Ω,ω) and 𝜖 > 0. For R > 1, define \(f_{R}:{\Omega }\to \mathbb {R}\) defined by fR(x) = f(x) if ∥x∥≤ R − 1, fR(x) = (R −∥x∥)f(x) if ∥x∥∈ [R − 1,R] and fR(x) = 0 if ∥x∥ > R. Then we have that \(f_{R}\in {W^{1}_{0}}({\Omega }_{R},\omega )\) and it is easy to check that there is an R0 > 0 such that for all R > R0, \(\|f-f_{R}\|_{W^{1}({\Omega },\omega )}<\epsilon \).

From now on, we fix R > R0. Applying Theorem III.2.11 in [19], conditions (i) and (ii) ensures that there is a sequence of functions \(f_{R,n}\in {W^{1}_{0}}({\Omega }_{R,n},\omega )\) such that \(\lim _{n\to \infty }\|f_{R}-f_{R,n}\|_{W^{1}({\Omega }_{R},\omega )}=0\).

And we conclude using (iii). □

Let now G be a connected metric space, and fix k ∈{1, 2}. Suppose that there is a locally finite covering \(({\Omega }_{\ell })_{\ell \in {\mathscr{L}}}\) of G, with open sets and and with \({\mathscr{L}}\) a countable set. Suppose also that this covering is such that for each , Ω is isometric to an open subset of Bm,k for some m ≥ 1. Suppose that there is a sequence of functions \((\varphi _{\ell })_{\ell \in {\mathscr{L}}}\) such that φ : G → [0, 1], φ = 0 on G ∖Ω, φ restricted to \({\Omega }_{\ell }^{i}\) is C1, with bounded derivatives, for each \(i\in \{1,\dots , m_{\ell }\}\) and such that \({\sum }_{\ell \in {\mathscr{L}}}\varphi _{\ell }=1\).

Let \(\omega :G\to [0,\infty ]\) be a measurable mapping. For each , then there is m ≥ 1 such that ΩBm,k, we let \(\omega _{\ell }:B_{m,k}\to [0,\infty ]\) be such that ω = ω on Ω. Suppose that \(\omega _{\ell }\in L^{1}_{loc}(B_{m,k})\) and \(\frac {1}{\omega _{\ell }}\in L^{1}_{loc}({\Omega }_{\ell })\).

Define W1(G,ω) to be the set of all measurable functions \(f:G\to \mathbb {R}\) such that for each there is \(f_{\ell }\in W^{1}({\Omega }_{\ell },\omega _{\ell })\) with f = f on Ω with \(\|f\|_{W^{1}(G,\omega )}<\infty \) where

$$\|f\|^{2}_{W^{1}(G,\omega)}={\int}_{G} \big(f^{2} + \|\nabla f\|^{2}\big)(x) \omega(x) dx.$$

Then W1(G,ω) equipped with the innner product

$$\langle f,g\rangle^{2}_{W^{1}(G,\omega)}={\int}_{G} \big(fg + \nabla f\cdot \nabla g \big)(x) \omega(x) dx$$

is a Hilbert space. Define also \({H^{1}_{0}}(G,\omega )\) as the completion of Cc(G) ∩ W1(G,ω). Note that if fW1(G,ω), we have that for each , \(f\varphi _{\ell }\in W^{1}({\Omega }_{\ell },\omega _{\ell })\).

Lemma A.3

Suppose that for all and all R > 0 there exists a non decreasing sequence of open subsets (Ω,R,n)n≥ 1 such that

  1. (i)

    \(\bigcup _{n\ge 1}{\Omega }_{\ell ,R,n}={\Omega }_{\ell ,R}\),

  2. (ii)

    \(\lim _{n\to \infty } \text {Cap}({\Omega }_{\ell ,R}\setminus \overline {\Omega }_{\ell ,R,n})=0\),

  3. (iii)

    for all n ≥ 1, \({W^{1}_{0}}({\Omega }_{\ell ,R,n},\omega _{\ell })={H^{1}_{0}}({\Omega }_{\ell ,R,n},\omega _{\ell })\).

Then we have \(W^{1}(G,\omega )={H^{1}_{0}}(G,\omega )\).

Proof

The proof of this lemma is almost identical to the one of Lemma A.2. We write \(f={\sum }_{\ell } f_{\ell }\), where f := fφ. For each 𝜖 > 0 there is a finite \({\mathscr{L}}_{0}\subset \mathcal L\) such that \(\|f-{\sum }_{\ell \in {\mathscr{L}}_{0}}f_{\ell }\|_{W^{1}(G,\omega )}<\epsilon \). And then we can follow the proof of Lemma A.2, for Ω = Ω and f = f and to find, for all \(\ell \in {\mathscr{L}}_{0}\), an integer n ≥ 1 and a function \(g_{\ell }\in C_{c}({\Omega }_{\ell ,R,n})\cap W^{1}({\Omega }_{\ell },\omega _{\ell })\) such that \(\|f_{\ell } -g_{\ell }\|_{W^{1}(G,\omega )}<\frac {\epsilon }{|{\mathscr{L}}_{0}|}\). Then \(\|f -{\sum }_{\ell \in {\mathscr{L}}_{0}} g_{\ell }\|_{W^{1}(G,\omega )}<2 \epsilon \). □

1.2 Regularity for Section ??

In this paragraph, we complete the proofs of Proposition 7.5 and of Proposition 7.6.

Let us first recall some notation of Section ??. The space \(\widetilde M=V\cup \bigcup _{i\in I} E_{i}\) is a metric graph, with Ei =]li,ri[×{i}. There is a continuous map \(\pi :\mathbb {R}^{2}\to \widetilde M\) such that if x ∈Ωi, π(x) = (H(x),i) ∈ Ei. For vV, \(I^{+}_{v}=\{i\in I, (l_{i},i)=v\}\), \(I^{-}_{v}=\{i\in I, (r_{i},i)= v\}\) and \(I_{v}=I^{+}_{v}\cup I^{-}_{v}\). Let d(v) be the cardinal of the set Iv, which is the degree of v in the graph \(\widetilde M\). Choose iIv and set hv = li if v = (li,i) or hv = ri if v = (ri,i). Remark that hv does not depend on the particular choice iIv. For vV, we define \(\gamma (v):=\bigcup _{(h,i)\sim v}\gamma _{i}(h)\), the connected level set associated to the vertex v. Then for all xγ(v), H(x) = hv.

The space \(\widetilde { {\mathscr{H}}}=\{f: f\circ \pi \in H^{1}(\mathbb {R}^{2})\}\) equipped with the inner product \(\langle f,g\rangle _{\widetilde H}:=\langle f\circ \pi ,g\circ \pi \rangle _{H^{1}(\mathbb {R}^{2})}\) is a Hilbert space. Let now \(f\in \widetilde {{\mathscr{H}}}\). For iI, let \(f_{i}:]l_{i},r_{i}[\to \mathbb {R}\) be defined by fi(h) = f(h,i). Then (see Lemma 7.4), for all iI, fi is weakly differentiable and we have

$$\|f\|^{2}_{\widetilde H} ={\sum}_{i\in I} {\int}_{l_{i}}^{r_{i}} (f^{\prime}_{i}(h))^{2} a_{i}(h) dh + {\sum}_{i\in I} {\int}_{l_{i}}^{r_{i}} (f_{i}(h))^{2} T_{i}(h) dh$$

where \(\displaystyle a_{i}(h)=\oint _{\gamma _{h,i}} |\nabla H| d\ell \) and \(\displaystyle T_{i}(h)=\oint _{\gamma _{h,i}} \frac {d\ell }{|\nabla H|}\). For \(\widetilde x=(h,i)\in E_{i}\), set \(a(\widetilde x)=a_{i}(h)\) and \(T(\widetilde x)=T_{i}(h)\).

In the following lemma we recall asymptotics given in chapter 8 in [9].

Lemma A.4

Let vV and iI(v). Then v = (hv,i) and as hhv, with (h,i) ∈ Ei,

  1. (1)

    If d(v) = 1,

    $$a_{i}(h)\sim a_{i,v} |h-h_{v}| \quad\text{ and }\quad T_{i}(h)\sim t_{i,v}$$
  2. (2)

    If d(v) ≥ 2,

    $$a_{i}(h)\sim a_{i,v} \quad\text{ and }\quad T_{i}(h)\sim t_{i,v} |\log|h-h_{v}||$$

where ai,v and ti,v are two finite positive constants.

Proof

By definition of v, either γ(v) contains exactly one extremum of H and d(v) = 1 or γ(v) contains a saddle point of H and d(v) ≥ 3 (d(v)≠ 2 since the stationary points of H are non-degenerate).

Suppose first that d(v) = 1. Then γ(v) = {x}, with x a local extremum of H. If H is quadratic at x then, the computation of \(a(\tilde x)\) and \(T(\tilde x)\) can easily be done explicitly and gives the stated asymptotics. In the general case, one can use an asymptotic expansion or the Morse Lemma to conclude.

Suppose now that d(v) ≥ 2. Let \(\tilde x=(h,i)\to v\). Since |∇H| is continuous and that γ(v) is compact with finite length, \(a(\tilde x)\) converges to a constant \(a_{i,v}=\oint _{\gamma _{h_{v},i}}|\nabla H|d \ell \) as \(\tilde x\to v\). The main contribution for T(h) comes when the orbit γh,i is near a stationary point xγ(v) because otherwise |∇H| is bounded away from 0. Note that there could be several such stationary points, each is a saddle. If H is quadratic in a small neighborhood \(\mathcal {V}\) of x then, the computation of an asymptotic can easily be done explicitly and gives \(\oint _{\gamma _{h,i}\cap \mathcal {V}}\frac 1{|\nabla H|}d \ell \sim C^{*}|\log |h-h_{v}||\) for some C > 0 which only depends on the number of times the orbit comes near x (one or two times since x is not degenerated) and on the Hessian matrix of H at x. In the general case, one can use the Morse Lemma to obtain the same asymptotics. Since each saddle point of γ(v) gives an asymptotic term of the same order, we obtain the stated result. □

Proof Proof of Proposition 7.5

Let \(f\in \widetilde {\mathcal H}\). Let \(v\in \mathcal {V}\) with d(v) ≥ 2, and iIv. Let Ai be an open set of ]li,ri[ such that \(\bar {A}_{i}\subset ]l_{i},r_{i}[\cup \{h_{v}\}\). Note that there is c > 0 such that Tiaic on Ai. Thus fi restricted to Ai belongs to H2(Ai) and as a consequence fi can be extended to a continuous function on ]li,ri[∪{hv}. Set Γi,v = ΩiH− 1(hv). Since fπ(x) = fiH(x), we have \(f_{|{\Gamma }_{i,v}}=f_{i}(h_{v})\).

Let now jIv with ji. Suppose first that Γi,v ∩Γj,v, then fi(hv) = fj(hv). If Γi,v ∩Γj,v = , then there is \((i_{0},i_{1},\dots ,i_{\ell })\in I_{v}^{\ell +1}\) such that i0 = i, i = j and for all 1 ≤ k, \({\Gamma }_{i_{k-1},v}\cap {\Gamma }_{i_{k},v}\ne \emptyset \) and so \(f_{i}(h_{v})=f_{i_{1}}(h_{v})={\dots } =f_{i_{\ell -1}}(h_{v})=f_{j}(h_{v})\). This proves that f is continuous on \(\widetilde M\setminus \mathcal {V}_{1}\), and completes the proof of the first implication of the Proposition 7.5.

Let now \(f:M\to \mathbb {R}\) be a measurable map such that f is continuous on \(\widetilde M\setminus \mathcal {V}_{1}\), fi is weakly differentiable for all iI and \(\|f\|_{\widetilde {\mathcal H}}<\infty \). Then, for all i, \(f\circ \pi \in H^{1}({\Omega }_{i})\) and fπ is continuous on M ∖Γ1, where Γ1 is the set of local extrema of H. Since Γ1 is a finite set, \(f\circ \pi \in H^{1}(\mathbb {R}^{2})\). This proves that \(f\in \widetilde {\mathcal H}\). □

Proof Proof of Proposition 7.6

To complete this proof, it remains to check that Assumption 6.9-(ii) is satisfied, i.e. that \(C_{c}(\widetilde M)\cap \widetilde {{\mathscr{H}}}\) is dense in \(\widetilde { {\mathscr{H}}}\).

Let i and j in I be such that d(Ei,Ej) = 0. Denote by vi,j the unique vertex in V such that vi,jEiEj. Set Ei,j := EiEj ∪{vi,j} and \(f_{i,j}=f_{|E_{i,j}}\). Let Ai,j be an open subset of Ei,j such that \(v_{i,j}\in A_{i,j}\Subset E_{i,j}\). Then (since there is c > 0 such that \(T(\widetilde x)\ge a(\widetilde x)\ge c\) in a neighborhood of v), fi,jH1(Ai,jEi) and ui,jH1(Ai,jEj). Since fi,j is continuous, we have that fi,j is weakly differentiable.

For each i, let ki :]li,ri[→]0,Li[ be defined such that ki(li) = 0 and \(dk_{i}(h)=\sqrt {\frac {T_{i}(h)}{a_{i}(h)}}dh\). Then ki is properly defined (in particular \(\displaystyle {\int \limits }_{l_{i}+} \sqrt {\frac {T_{i}(h)}{a_{i}(h)}}dh <\infty \) and \(\displaystyle L_{i}={\int \limits }_{l_{i}}^{r_{i}} \sqrt {\frac {T_{i}(h)}{a_{i}(h)}}dh <\infty \) only for the unique i for which Ωi is unbounded). Define a new metric graph with edges \({E^{G}_{i}}:=]0,L_{i}[\times \{i\}\) with the same adjacency rules as for \(\widetilde M\). Denote this new metric graph by G. By abuse of notation, The set of vertices of G will also be denoted by V.

For vV, let I(v) be the set of all i such that \({E^{G}_{i}}\) is an edge adjacent to v. For iI(v), set \({O^{i}_{v}}:= \{x \in {E^{G}_{i}}: d(x,v)<\frac {2L_{i}}{3}\}\) and \(O_{v}:=\{v\}\cup \cup _{i\in I(v)}{O^{i}_{v}}\). When d(v) = 1 set \({O^{0}_{v}}=O_{v}\setminus \{v\}\) and when d(v) ≥ 2, set \({O^{0}_{v}}=O_{v}\). Then \({O^{0}_{v}}\) is on open subset of Bm,1 with m = d(v) = |I(v)|.

Let G0 be the metric graph obtained out of G by taking out of G the vertices of degree 1, and denote by V0 the set of vertices of G0. Then it is easy to check that \(({O^{0}_{v}})_{v\in V^{0}}\) is a covering of G0 and that there are functions φv, vV0, such that φv : G0 → [0, 1], φv = 0 on \(G\setminus {O^{0}_{v}}\), φv restricted to \({O^{0}_{v}}\) is C1, with bounded derivatives, for each \(i\in \{1,\dots , d(v)\}\) and such that \({\sum }_{v\in V^{0}}\varphi _{v}=1\).

For \(f\in \widetilde {{\mathscr{H}}}\), let \(g:G^{0}\to \mathbb {R}\) be defined by g(v) = f(v) if vV0 and such that g(ki(h),i) = f(h,i) if (h,i) ∈ Ei.

Let ij such that \(d({E^{G}_{i}},{E^{G}_{j}})=0\) and set \(E^{G}_{i,j}:={E^{G}_{i}}\cup {E^{G}_{j}}\cup \{x_{i,j}\}\), we then have that \(g_{i,j}:=g_{|E^{G}_{i,j}}\) is weakly differentiable. This holds because fi,j is weakly differentiable and gi,j = fi,jhi,j, where \(h_{i,j}:E^{G}_{i,j}\to E_{i,j}\) is the continuous function, differentiable on \({E^{G}_{i}}\) and on \({E^{G}_{j}}\), defined by \(h_{i,j}(k,i)=(k_{i}^{-1}(k),i)\) if \((k,i)\in {E^{G}_{i}}\), \(h_{i,j}(k,j)=(k_{j}^{-1}(k),j)\) if \((k,j)\in {E^{G}_{j}}\) and hi,j(xi,j) = xi,j.

Then we have that \(f\in \widetilde {{\mathscr{H}}}\) if and only if g defined above belongs to W(G0,ω) with ω a measurable function on G0 such that if \((k,i)\in {E^{G}_{i}}\), \(\omega (k,i)=\omega _{i}(k)=\sqrt {a_{i}T_{i}}\circ k^{-1}_{i}(k)\).

Lemma A.5

For \(\widetilde x\in G^{0}\) and vV. We have as \(\widetilde x\to v\),

  1. (1)

    If d(v) = 1, \(\omega (\widetilde x)\sim c_{v} d(\widetilde x,v)\)

  2. (2)

    If d(v) ≥ 2, \(\omega (\widetilde x)\sim c_{v}\sqrt {|\log (d(\widetilde x,v))|} \to \infty \)

where cv > 0.

Proof

The case d(v) = 1 is straightforward. For the case d(v) ≥ 2, we let \(\widetilde x=(k,i)\in G^{0}\) and assume v = (0,i), then \(k=d(\widetilde x,v)\) and \(h:=k^{-1}_{i}(k)\) with \(k_{i}^{\prime }(h)=\sqrt {\frac {T_{i}(h)}{a_{i}(h)}}\sim C_{v}\sqrt {|\log h|}\) as h → 0. An integration by part yields that \(k(h)\sim C h\sqrt {|\log h|}\) and thus \(|\log (k)|\sim |\log h|\). Then, since \(\omega (\widetilde x)=\sqrt {a_{i}T_{i}}(h)\sim C\sqrt {|\log h|}\), we get the result. The same result holds if v = (Li,i). □

To prove that \(C_{c}(\widetilde M)\cap \widetilde {{\mathscr{H}}}\) is dense in \(\widetilde {{\mathscr{H}}}\), we will now use Lemma A.3.

Fix R > 0. For vV and n ≥ 1, set Ov,R,n = Ov,R if d(v) ≥ 2 and set Ov,R,n := {xOv,R : d(x,v) > n− 1} if d(v) = 1. Then (i) of Lemma A.3 is satisfied.

Let us now check that (ii) is satisfied. This has only to be checked for vV with d(v) = 1. Let \({E^{G}_{i}}\) be the unique edge adjacent to v. Without loss of generality we will suppose that v = (0,i). We then have that Ov,ROv,R,n =]0,n− 1[×{i}. To prove that Cap(Ov,ROv,R,n) → 0. In this case, fix A > 0 such that A < R and \(A<\frac {2L_{i}}{3}\) and take (for n > A− 1), gn(k) = 1 if kn− 1 and gn(k) = 0 if k > A and \(g_{n}(k)=\frac {\log (A)-\log (k)}{\log (A)-\log (n^{-1})}\) if n− 1kA. Then \(Cap(O_{v,R}\setminus O_{v,R,n})\le {\int \limits }_{n^{-1}}^{A} \frac {\omega _{i}(k)}{k^{2}(\log (A)+\log (n))^{2}} dk + {\int \limits }_{n^{-1}}^{A} \frac {(\log (A)-\log (k))^{2}\omega _{i}(k)}{(\log (A)+\log (n))^{2}} dk\) which converges to 0 as \(n\to \infty \). This proves (ii)

To check (iii), we shall use Lemma A.1. for the open sets Ov,R,nBm,1, with m = d(v). For each v,R,n, we define an extension \(\bar {\omega }\) of \(\omega _{|O_{v,R,n}}\), defined on Bm,1. By a slight abuse of notation, we set identify v as 0 in the definition of Bm,1 and keep the labels of I(v) to identify the branches of Bm,1.

If m = d(v) = 1 then recall that \(B_{1,1}=\mathbb {R}\). Ov,R,n is an open interval ]l,r[ and we let \(\bar {\omega }(k)=\omega (l)\in ]0,\infty [\) if kl and \(\bar {\omega }(k)=\omega (r)\in ]0,\infty [\) if k > r. The \(\bar {\omega }\) is continuous and positive, and it thus belongs to the class A2.

If m = d(v) ≥ 2, for iI+(v), \(O_{v,R,n}\cap {E^{G}_{i}}\) is an open interval ]0,r[ (with r = (2Li/3) ∧ R) and for k > r, we set \(\bar {\omega }(k,i)=\omega (r,i)\in ]0,\infty [\) and \(\bar {\omega }(v)=\infty \). For iI(v), \(O_{v,R,n}\cap {E^{G}_{i}}\) is an open interval ]l,Li[ (with l = Li − (2Li/3) ∧ R) and we set

$$ \bar{\omega}(k,i)= \begin{cases} \omega(L_{i}-k,i)\in ]0,\infty[&\text{ for }0<k<L_{i}-l \\ \omega(g,i)\in ]0,\infty[&\text{ for }k>L_{i}-l \end{cases} $$

and \(\bar {\omega }(v)=\infty \). This defines \(\bar {\omega }\) on Bm,1. It remains to check that for ij in I(v), we have that \(\bar {\omega }_{i,j}\) belongs to the class A2. Note that \(\lim _{x\to v}\bar {\omega }_{i,j}(x)=\infty \). Since \(\omega _{i,j}(x)\sim c \sqrt {\log (k)}\) as k := d(x,v) → 0, it is a simple exercise to show that \(\bar {\omega }_{i,j}\) belongs to the class A2.

Then Lemma A.3 can be applied. This proves that \(W(G^{0},\omega )={H^{1}_{0}}(G^{0},\omega )\). Let \(f\in \widetilde {{\mathscr{H}}}\) and gW(G0,ω) defined as above by \(g(k,i)=f(k_{i}^{-1}(k),i)\) if \((k,i)\in {E^{G}_{i}}\) and g(v) = f(v) if vG0. Since \(W(G^{0},\omega )={H^{1}_{0}}(G^{0},\omega )\), for all 𝜖 > 0 there is g𝜖Cc(G0) ∩ W(G0,ω) such that \(\|g-g_{\epsilon }\|_{W(G^{0},\omega )}<\epsilon \). Set f𝜖(h,i) = g𝜖(ki(h),i) if (h,i) ∈ Ei, f𝜖(v) = g𝜖(v) if vV0 and f𝜖(v) = 0 if vVV0. Then \(f_{\epsilon }\in C_{c}(\widetilde M)\cap \widetilde {{\mathscr{H}}}\) and \(\|f-f_{\epsilon }\|_{\widetilde {{\mathscr{H}}}}=\|g-g_{\epsilon }\|_{W(G^{0},\omega )}<\epsilon \). □

1.3 Regularity for Section ??

To prove the regularity we can apply directly Lemma A.2 since our state space will already be an open subset of B4,2. Set I := {1, 2, 3, 4}. Recall that \(\widetilde {M}=\cup _{i\in I} C_{i}\) is constituted of four cones glued along one edge. Set \(\widetilde M_{0}:=\{y\in \widetilde M: y_{1}y_{2} > 0\}\).

1.3.1 The Space \(\widetilde {\mathcal H}\)

Recall that for t ∈]0, 1[

$$ \begin{array}{@{}rcl@{}} K(t)&=&{\int}_{0}^{\pi/2}\frac{d\theta}{\sqrt{1-t^{2}\sin^{2}\theta}}, E(t)={\int}_{0}^{\pi/2} \sqrt{1-t^{2}\sin^{2}\theta} d\theta \end{array} $$
(A.3)
$$ \begin{array}{@{}rcl@{}} \alpha(t)&=&1-\frac{E(t)}{K(t)} \qquad\quad\text{ and } \lambda(t)=1-\frac{\alpha(t)(1+t^{2})}{2t^{2}}. \end{array} $$
(A.4)

For y ∈∪iIC̈i, let \(t(y)=\frac {|y_{1}|\wedge |y_{2}|}{|y_{1}|\vee |y_{2}|}\). Then \(t(y)=\frac {y_{2}}{y_{1}}\) for yC̈1C̈3 and \(t(y)=\frac {y_{1}}{y_{2}}\) for yC̈2C̈4.

For iI, set \(\widetilde {m}_{i}\) the measure on Ci with density hi(y)eW(∥y∥) with respect to Lebesgue measure on Ci, where

$$ \begin{array}{@{}rcl@{}} h_{1}(y)=h_{3}(y)&=4\sqrt{2} |y_{2}|K\left( t(y)\right); h_{2}(y)=h_{4}(y)=4\sqrt{2} |y_{1}|K\left( t(y)\right). \end{array} $$
(A.5)

For y ∈∪iIC̈i, set a(y) := νy(∇π ⊗∇π). We have that \(\nu _{y}({x_{3}^{2}})=2{y_{1}^{2}}\alpha (t)\) for yC̈1, then,

$$ \begin{array}{@{}rcl@{}} a(y)&=&\begin{pmatrix} 1&0\\ 0&1 \end{pmatrix} +\frac{\alpha(t)}{2t^{2}}\begin{pmatrix} -t^{2}&t\\ t&-1 \end{pmatrix} \text{ if } y\in \mathring{C}_{1}\cup \mathring{C}_{3} \text{ and } t=\frac{y_{2}}{y_{1}};\\ a(y)&=&\begin{pmatrix} 1&0\\ 0&1 \end{pmatrix} +\frac{\alpha(t)}{2t^{2}}\begin{pmatrix} -1&t\\ t&-t^{2} \end{pmatrix} \text{ if } y\in \mathring{C}_{2}\cup \mathring{C}_{4} \text{ and } t=\frac{y_{1}}{y_{2}}. \end{array} $$
(A.6)

The eigenvalues of a(y) are 1 and λ(t), with corresponding eigenvectors (y1,y2) and (−y2,y1). If iI and yC̈i, set ai(y) = a(y).

Let us now collect some asymptotics.

Lemma A.6

As t → 0+,

$$ \begin{array}{@{}rcl@{}} E(t)&=&\frac{\pi}{2}\left( 1-\frac{t^{2}}{4}+O(t^{4})\right); K(t)=\frac{\pi}{2}\left( 1+\frac{t^{2}}{4}+O(t^{4})\right); \end{array} $$
(A.7)
$$ \begin{array}{@{}rcl@{}} \alpha(t)&=&\frac{t^{2}}{2}\left( 1+O(t^{2})\right); \lambda(t)=\frac{3}{4}+O(t^{2}). \end{array} $$
(A.8)

As t → 1,

$$ \begin{array}{@{}rcl@{}} E(t)&=&1+o(1); K(t)=-\frac{1}{2}\log(1-t) + O(1); \end{array} $$
(A.9)
$$ \begin{array}{@{}rcl@{}} \alpha(t)&=&1+\frac{2}{\log(1-t)}(1+o(1)); \lambda(t)=\frac{-2}{\log(1-t)}(1+o(1)). \end{array} $$
(A.10)

Proof

Asymptotics for E and K are standard, the others are simple consequences. □

Let \(f\in \widetilde {{\mathscr{H}}}\). For iI, the map \(f_{i}:=f_{|C_{i}}\) is weakly differentiable on Ci and, using the change of variable formula Eq. ??, we have that

$$ \|f\|^{2}_{\widetilde{\mathcal{H}}}={\sum}_{i=1}^{4}{\int}_{C_{i}}f^{2} d\widetilde{m}_{i}+{\sum}_{i=1}^{4}{\int}_{C_{i}}a_{i}^{k\ell}\partial_{k} f_{i}\partial_{\ell} f_{i}d\widetilde{m}_{i}. $$
(A.11)

Set \({\Omega }:=]0,+\infty [\times B_{4,1}\subset B_{4,2}\), Ωi := Ω ∩ Ei for iI and Ωi,j := Ω ∩ Ei,j for ijI. Let us remark from Lemma A.6 that \(\displaystyle \mathcal {I}:={{\int \limits }_{0}^{1}} \frac {du}{(1+u^{2})\sqrt {\lambda (u)}} <+\infty \). Set \(c=\frac {\pi }{2\mathcal {I}}\) and define \(\varphi :[0,1]\to [0,\frac {\pi }2]\) by

$$ \varphi(t)=c {{\int}_{t}^{1}} \frac{du}{(1+u^{2})\sqrt{\lambda(u)}}. $$
(A.12)

Define \(z:\cup _{i=1}^{4} \mathring {C}_{i} \to ]0,+\infty [^{2}\) by

$$ \begin{array}{@{}rcl@{}} z(y)&=|y|\left( \cos\varphi (t(y)),\sin \varphi\left( t(y)\right)\right). \end{array} $$
(A.13)

Note that for all iI, \(z_{i}= z_{|\mathring {C}_{i}}\) is one to one.

For \(g:{\Omega }\to \mathbb {R}\) define \(\mathcal F(g):\widetilde {M}_{0}\to \mathbb {R}\) by \(\mathcal F(g)(y)=g(z(y),i)\) if yC̈i and \(\mathcal {F}(g)(y)=g(y,0)\) if yD. For \(f:\widetilde {M}\to \mathbb {R}\) (or for \(f:\widetilde {M}_{0}\to \mathbb {R}\)), define \(\mathcal G(f):{\Omega }\to \mathbb {R}\) such that if (z,i) ∈Ωi, then \(\mathcal {G}(f)(z,i)=f_{i}\circ z_{i}^{-1}(z)\) and if z > 0, then \(\mathcal G(f)(z,0)=f(z)\). Then, \(\mathcal G\circ \mathcal F(g)=g\) and we have that gCc(Ω) if and only if \(\mathcal {F}(g)\in C_{c}(\widetilde {M}_{0})\).

Define \(k:[0,\frac {\pi }{2}[\to \mathbb {R}^{+}\cup \{\infty \}\) by \(k(0)=\infty \) and, for \(\phi \in ]0,\frac {\pi }{2}[\) and t = φ− 1(ϕ),

$$ k(\phi):=\frac{4\sqrt{2}tK(t)\sqrt{\lambda(t)}}{c\sqrt{1+t^{2}}}. $$
(A.14)

For \(z=r({\cos \limits } \phi ,{\sin \limits } \phi )\) with \((r,\phi )\in ]0,\infty [\times [0,\frac {\pi }2[\), set ω(z,i) := reW(r)k(ϕ) and

$$ b(z,i):=\begin{pmatrix} \cos^{2}\phi+c^{2}\sin^{2}\phi & (1-c^{2})\sin\phi\cos\phi\\ (1-c^{2})\sin\phi\cos\phi & \sin^{2}\phi+c^{2}\cos^{2}\phi \end{pmatrix} =R_{\phi} \begin{pmatrix} 1 & 0\\ 0 & c^{2} \end{pmatrix} R_{-\phi}$$

where Rϕ is the matrix associated to the rotation of angle ϕ,

$$R_{\phi}= \begin{pmatrix} \cos\phi&-\sin\phi\\ \sin\phi&\cos\phi \end{pmatrix}.$$

Let \(f\in \widetilde {\mathcal H}\) and \(g=\mathcal G(f)\). The change of variable z = zi(y) on each Ci yields

$$ \begin{array}{@{}rcl@{}} \|f\|^{2}_{\widetilde{\mathcal{H}}}={\sum}_{i=1}^{4}{\int}_{{\Omega}_{i}}g^{2}(z,i)\omega(z,i)dz+{\sum}_{i=1}^{4}{\int}_{{\Omega}_{i}}b^{k\ell}(z,i)\partial_{k} g(z,i)\partial_{\ell} g(z,i)\omega(z,i)dz. \end{array} $$
(A.15)

Recall that the norm on W1(Ω,ω) is given by

$$ \|g\|^{2}_{W^{1}({\Omega},\omega)}={\sum}_{i=1}^{4}{\int}_{{\Omega}_{i}}(g^{2}(z,i)+|\nabla g(z,i)|^{2})\omega(z,i)dz. $$
(A.16)

We thus get that

$$ (1\wedge c^{2})\|g\|^{2}_{W^{1}({\Omega},\omega)} \leq \|f\|^{2}_{\widetilde{\mathcal{H}}} \leq(1\vee c^{2})\|g\|^{2}_{W^{1}({\Omega},\omega)}. $$
(A.17)

Lemma A.7

\(f\in \widetilde {{\mathscr{H}}}\) if and only if \(\mathcal G(f)\in W^{1}({\Omega },\omega )\).

Proof

In this proof, we fix \(f:\widetilde M\to \mathbb {R}\) and we set \(g:=\mathcal G(f)\). If \(f \in \widetilde H\), then for each i, fi is weakly differentiable on C̈i and \(g_{|{\Omega }_{i}}\) is weakly differentiable on Ωi. And Eq. A.17 show that \(g_{i}:=g_{|{\Omega }_{i}}\in W^{1}({\Omega }_{i},\omega _{i})\), where \(\omega _{i}=\omega _{|{\Omega }_{i}}\). Moreover, if \(\mathcal {O}\) be a bounded open set in Ωi,j, then \(g_{|\mathcal {O}_{i}}\in H^{1}(\mathcal {O}_{i})\), \(g_{|\mathcal {O}_{j}}\in H^{1}(\mathcal {O}_{j})\) and coincide on \(\mathcal {O}\cap E_{0}\). Since ω is bounded from below on \(\mathcal {O}\), this implies that \(g_{|\mathcal {O}}\in H^{1}(\mathcal {O})\supset W^{1}(\mathcal {O},\omega _{i,j})\). We thus have that \(g_{|{\Omega }_{i,j}}\in W^{1}({\Omega }_{i,j},\omega _{i,j})\), which proves that gW1(Ω,ω).

On the converse, if gW1(Ω,ω), then using that \(f\circ \pi =\mathcal {F}(g)\circ \pi \) and Eq. A.17, we have fπH1(m) and so \(f\in \widetilde H\). □

Lemma A.8

\({H^{1}_{0}}({\Omega },\omega )=W^{1}({\Omega },\omega )\)

We use Lemma A.2 to prove this lemma.

This lemma shows the density of \(C_{c}(\widetilde M)\cap \widetilde {{\mathscr{H}}}\) in \(\widetilde {{\mathscr{H}}}\). Indeed, \(\mathcal {F}:\widetilde H\to W^{1}({\Omega },\omega )\) and \(\mathcal {G}: W^{1}({\Omega },\omega )\) are continuous mappings and if gCc(Ω) ∩ W1(Ω,ω) then \(\mathcal {F}(g)\in C_{c}(\widetilde M)\cap \widetilde H\). Therefore, if \(f\in \widetilde H\) and \(g=\mathcal {G}(f)\), there is a sequence gnCc(Ω) ∩ W1(Ω,ω) approximating g in W1(Ω,ω) and so \(f_{n}:=\mathcal {F}(g_{n})\) is a sequence in \(C_{c}(\widetilde M)\cap \widetilde H\) approximating f in \(\widetilde H\).

Proof Proof of Lemma A.8

For R > 0, recall that ΩR = {z ∈Ω : ∥z∥ < R} and define, for n ≥ 1, \({\Omega }_{R,n}:=\{(r{\cos \limits } \phi , r{\sin \limits } \phi ,i) : (r,\phi ,i)\in ]0,R[\times [0,\frac \pi 2-\frac 1n[\times I\}\).

In order to apply Lemma A.2, we have to check that (i), (ii) and (iii) are satisfied. Item (i) is clearly satisfied.

To prove (ii), we construct a sequence of non negative functions (gn) on Ω, such that gn(z) = 1 in ΩR,n ∖ΩR and \(\lim _{n\to +\infty }\|g_{n}\|^{2}_{W^{1}({\Omega },\omega )}=0\). Let us first obtain asymptotics for ω.

Lemma A.9

We have, as \(\phi \to \frac {\pi }{2}^{-}\), \( k(\phi )\sim \frac {3\pi }{8c^{2}}(\frac {\pi }{2}-\phi ). \)

Proof

Using Lemma A.6 and Eq. A.12, we have that \(\frac {\pi }{2}-\varphi (t)\sim \frac {2c}{\sqrt {3}}t\) as t → 0+. Thus, since ϕ = φ(t), we have \(t\sim \frac {\sqrt {3}}{2c}(\frac {\pi }{2}-\phi )\) as \(\phi \to \frac {\pi }{2}^{-}\). From Lemma A.6 and Eq. A.14, we obtain \(k(\phi )\sim \frac {\pi \sqrt {3}}{4c}t\sim \frac {3\pi }{8c^{2}}(\frac {\pi }{2}-\phi )\) as \(\phi \to \frac {\pi }{2}^{-}\). □

Using this Lemma, we can fix \(0<\phi _{0}<\frac \pi 2\) and c0 > 0 such that \(0\leq k(\phi )\leq c_{0}(\frac {\pi }{2}-\phi )\) for all \(\phi \in [\phi _{0},\frac {\pi }{2}]\). Then we define \(f_{1}:]0,+\infty [\to \mathbb {R}\), \(f_{2,n}:[0,\frac {\pi }2[\to \mathbb {R}\) by

$$ \begin{array}{@{}rcl@{}} f_{1}(r)&=& \begin{cases} 1&\text{ for }r<R \\ 2-\frac{r}{R}&\text{ for }R\leq r<2R \\ 0&\text{ for }2R\leq r \end{cases}, \end{array} $$
(A.18)
$$ \begin{array}{@{}rcl@{}} f_{2,n}(\phi)&=& \begin{cases} 0&\text{ for }0\leq \phi\leq \phi_{0} \\ \frac{\ln(\frac{\pi}{2}-\phi)-\ln(\frac{\pi}{2}-\phi_{0})}{\ln(\frac1n)-\ln(\frac{\pi}{2}-\phi_{0})}&\text{ for }\phi_{0}\leq\phi<\frac{\pi}{2}-\frac1n \\ 1&\text{ for }\frac{\pi}{2}-\frac1n\le \phi \end{cases} \end{array} $$
(A.19)

For iI and \(z=r(\cos \limits (\phi ),\sin \limits (\phi ))\in ]0,+\infty [^{2}\), set gn(z,i) = f1(r)f2,n(ϕ). We then have

$$ \|g_{n}\|^{2}_{W^{1}({\Omega},\omega)}=4{\int}_{{\Omega}_{1}}({g_{n}^{2}}(z)+|\nabla g_{n}(z)|^{2})\omega(z)dz. $$
(A.20)

Since \(|\nabla g_{n}(z)|^{2}=(f_{1}^{\prime })^{2}(r)f^{2}_{2,n}(\phi )+\frac 1{r^{2}}{f^{2}_{1}}(r)(f^{\prime }_{2,n})^{2}(\phi )\), we have

$$ \begin{array}{@{}rcl@{}} {\int}_{{\Omega}_{1}}{g_{n}^{2}}(z)\omega(z)dz &=&{\int}_{0}^{2R}{f^{2}_{1}}(r)r^{2}e^{-W(r)}dr{\int}_{\phi_{0}}^{\frac{\pi}{2}}f^{2}_{2,n}(\phi)k(\phi)d\phi, \end{array} $$
(A.21)
$$ \begin{array}{@{}rcl@{}} {\int}_{{\Omega}_{1}}|\nabla g_{n}(z)|^{2}\omega(z)dz &=&{\int}_{0}^{2R}(f^{\prime}_{1})^{2}(r)r^{2}e^{-W(r)}dr{\int}_{\phi_{0}}^{\frac{\pi}{2}}f^{2}_{2,n}(\phi)k(\phi)d\phi\\ &&+{\int}_{0}^{2R}{f^{2}_{1}}(r)e^{-W(r)}dr{\int}_{\phi_{0}}^{\frac{\pi}{2}}(f^{\prime}_{2,n}(\phi))^{2}k(\phi)d\phi. \end{array} $$
(A.22)

The integrals involving f1 are finite and does not depend on n. For f2,n, we have

$$ \begin{array}{@{}rcl@{}} {\int}_{\phi_{0}}^{\frac{\pi}{2}}f^{2}_{2,n}(\phi)k(\phi)d\phi &\leq& c_{0}{\int}_{\phi_{0}}^{\frac{\pi}{2}}f^{2}_{2,n}(\phi)\left( \frac{\pi}{2}-\phi\right) d\phi \end{array} $$
(A.23)
$$ \begin{array}{@{}rcl@{}} &=& c_{0}\left( {\int}_{\phi_{0}}^{\frac{\pi}{2}-\frac{1}{n}}f^{2}_{2,n}(\phi)\left( \frac{\pi}{2}-\phi\right) d\phi + {\int}_{\frac{\pi}{2}-\frac{1}{n}}^{\frac{\pi}{2}}\left( \frac{\pi}{2}-\phi\right) d\phi\right) \end{array} $$
(A.24)
$$ \begin{array}{@{}rcl@{}} &\leq& c_{0}\left( {\int}_{\phi_{0}}^{\frac{\pi}{2}}\left( \frac{\ln(\frac{\pi}{2}-\phi)-\ln(\frac{\pi}{2}-\phi_{0})}{\ln(\frac{1}{n})-\ln(\frac{\pi}{2}-\phi_{0})}\right)^{2} \left( \frac{\pi}{2}-\phi\right) d\phi + \frac1{2n^{2}}\right).\\ \end{array} $$
(A.25)

Thus, \(\lim _{n\to +\infty }{\int \limits }_{\phi _{0}}^{\frac {\pi }{2}}f^{2}_{2,n}(\phi )k(\phi )d\phi =0\). We also have

$$ \begin{array}{@{}rcl@{}} {\int}_{\phi_{0}}^{\frac{\pi}{2}}(f^{\prime}_{2,n}(\phi))^{2}k(\phi)d\phi &\leq& c_{0}{\int}_{\phi_{0}}^{\frac{\pi}{2}-\frac{1}{n}}(f^{\prime}_{2,n})^{2}(\phi)\left( \frac{\pi}{2}-\phi\right) d\phi \end{array} $$
(A.26)
$$ \begin{array}{@{}rcl@{}} &=&\frac{c_{0}}{(\ln(n^{-1})-\ln(\frac{\pi}{2}-\phi_{0}))^{2}}{\int}_{\phi_{0}}^{\frac{\pi}{2}-\frac{1}{n}}\frac{d\phi}{\frac{\pi}{2}-\phi} \end{array} $$
(A.27)
$$ \begin{array}{@{}rcl@{}} &\leq&\frac{c_{0}}{|\ln(n^{-1})-\ln(\frac{\pi}{2}-\phi_{0})|}. \end{array} $$
(A.28)

Thus, \(\lim _{n\to +\infty } {\int \limits }_{\phi _{0}}^{\frac {\pi }{2}}(f^{\prime }_{2,n}(\phi ))^{2}k(\phi )d\phi =0\) and then \(\lim _{n\to +\infty }\|g_{n}\|^{2}_{W^{1}({\Omega },\omega )}=0\). Item (ii) is proven.

To prove (iii), we apply Lemma A.1. Let us first define an extension \(\bar {\omega }_{n,R}\) to B4,2 of ω which coincide with ω on ΩR,n. We define \(k_{n}:[-\pi ,\pi ]\to \mathbb {R}^{+}\cup \{\infty \}\) by

$$ k_{n}(\phi)= \begin{cases} k(|\phi|)&\text{ for } \phi\in [-\frac\pi2+\frac1n,\frac\pi2-\frac1n]\\ k(\frac\pi2-\frac1n)&\text{ elsewhere,} \end{cases} $$
(A.29)

and define the weight \(\bar {\omega }_{n,R}:B_{4,2}\to \mathbb {R}^{+}\) by \(\bar {\omega }_{n,R}(z,i):=re^{-W(r\wedge R)}k_{n}(\phi )\) where \(z=(r\cos \limits \phi , r\sin \limits \phi )\) with \((r,\phi ,i)\in \mathbb {R}^{+}\times [-\pi ,\pi ]\times I\). Then, \(\bar {\omega }_{n,R}\) coincide with ω on ΩR,n. Note that for ij, the restriction of \(\bar {\omega }_{n,R}\) to Ei,j (identified with \(\mathbb {R}^{2}\)) does not depend on i and j. Denote this common measure ωn,R and set \(\omega _{n}:\mathbb {R}^{2}\to \mathbb {R}^{+}\cup \{+\infty \}\), defined by ωn(z) = rkn(ϕ) where \(z=r(\cos \limits \phi ,\sin \limits \phi )\in \mathbb {R}^{2}\). Since, \(0<\inf _{r>0} e^{-W(r\wedge R)} \le \sup _{r>0}e^{W(r\wedge R)}\), ωn,R belongs to the class A2 if and only if ωn belongs to the class A2.

Define for ρ > 0, \(z\in \mathbb {R}^{2}\),

$$C(z,\rho)=\frac1{\pi^{2} \rho^{4}}{\int}_{B(z,\rho)}\omega_{n}{\int}_{B(z,\rho)}\frac1{\omega_{n}}.$$

where B(z,ρ) is the ball at center z and radius ρ.

Let us first obtain asymptotics for ωn.

Lemma A.10

We have, as ϕ → 0 +, \( k(\phi )\sim \frac 1{2c}\left |\log \left (\phi \right )\right |^{1/2}. \)

Proof

For ϕ > 0 close to 0 and t = φ− 1(ϕ), set δ := 1 − t. We have

$$ \phi=c{\int}_{1-\delta}^{1}\frac{du}{(1+u^{2})\sqrt{\lambda(u)}}=c{\int}_{0}^{\delta}\frac{du}{(1+(1-u)^{2})\sqrt{\lambda(1-u)}} $$
(A.30)

As u → 0+, using Lemma A.6, we get \(\sqrt {\lambda (1-u)}\sim \frac {\sqrt {2}}{\sqrt {|\ln (u)|}}\), and thus as ϕ → 0+, by integration by parts

$$ \begin{array}{@{}rcl@{}} \phi\sim \frac{c}{2\sqrt{2}}{\int}_{0}^{\delta}\sqrt{|\log(u)|}du\sim\frac{c}{2\sqrt{2}}\delta\sqrt{|\log(\delta)|}. \end{array} $$
(A.31)

This entails that \(\log (\delta )\sim \log (\phi ).\) It is straightforward, using Lemma A.6 again, that \(K(t)\sqrt {\lambda (t)}\sim \frac 1{\sqrt {2}}\sqrt {|\log (\delta )|}\). Then we obtain that \(k(\phi )\sim \frac 1{2c}\sqrt {|\log (\delta )|} \sim \frac 1{2c}\sqrt {|\log (\phi )|}\). □

Let us now prove the condition A2 for ωn.

Lemma A.11

ωn satisfies the A2-Muckenhoupt condition: \(\sup _{z,\rho }C(z,\rho )<+\infty \).

Proof Proof of Lemma A.11

Note that kn is a continuous positive function on [−π,π] ∖{0}. Lemma A.10 shows that \(k_{n}(\phi )\sim \frac 1{2c}\sqrt {\left |\log |\phi |\right |}(1+o(1))\) as ϕ → 0. Thus the integrals \({\int \limits }_{-\pi }^{\pi }k_{n}(\phi )d\phi \) and \({\int \limits }_{-\pi }^{\pi }k_{n}(\phi )^{-1}d\phi \) are finite and ωn and \(\omega _{n}^{-1}\) are locally integrable on \(\mathbb {R}^{2}\). Therefore (z,ρ)↦C(z,ρ) is a continuous function. Note also that for λ > 0 and \(z\in \mathbb {R}^{2}\), ωn(λz) = λωn(z), then C(λz,λρ) = C(z,ρ). This leaves two cases to investigate: z = 0, ρ = 1 or |z| = 1, ρ > 0.

For z = 0, ρ = 1, we have

$$ \begin{array}{@{}rcl@{}} C(0,1)=\frac1{\pi^{2}}{\int}_{B(0,1)}\omega_{n}{\int}_{B(0,1)}\frac1{\omega_{n}} =\frac1{3\pi^{2}}\displaystyle{\int}_{-\pi}^{\pi}k_{n}{\int}_{-\pi}^{\pi}\frac1{k_{n}}<+\infty. \end{array} $$
(A.32)

Obviously, \(C(0,\rho )=C(0,1)=C_{n}<+\infty \) does not depend on ρ.

For the second case, fix ρ > 0 and \(z=(\cos \limits \phi ,\sin \limits \phi )\), with ϕ ∈ [−π,π]. If \(\rho \geq \frac {1}{2}\), since B(z,ρ) ⊂ B(0,ρ + 1), we get \(C(z,\rho )\leq \frac {(\rho +1)^{4}}{\rho ^{4}}C_{n}\le 3^{4} C_{n}\). If \(\rho <\frac 12\), we set \(\theta =\arcsin (\rho )\in ]0,\frac {\pi }{6}[\). We have \(B(z,\rho )\subset T(z,\rho ):=\{z=r(\cos \limits \psi ,\sin \limits \psi ); (r,\psi )\in [1-\rho ,1+\rho ]\times [\phi -\theta ,\phi +\theta ]\}\). We obtain

$$ \begin{array}{@{}rcl@{}} {\int}_{B(z,\rho)}\omega_{n} \leq{\int}_{T(z,\rho)}\omega_{n} &=&{\int}_{1-\rho}^{1+\rho} r^{2}dr{\int}_{\phi-\theta}^{\phi+\theta}k_{n} =\frac23(3\rho+\rho^{3}){\int}_{\phi-\theta}^{\phi+\theta}k_{n}, \end{array} $$
(A.33)
$$ \begin{array}{@{}rcl@{}} {\int}_{B(z,\rho)}\frac1{\omega_{n}} \leq{\int}_{T(z,\rho)}\frac1{\omega_{n}} &=&2\rho{\int}_{\phi-\theta}^{\phi+\theta}\frac1{k_{n}}. \end{array} $$
(A.34)

Then we get (setting \(c_{0}=\frac {13}{3\pi ^{2}}\))

$$ C(z,\rho)\leq \frac{4(3+\rho^{2})}{3\pi^{2}\rho^{2}}{\int}_{\phi-\theta}^{\phi+\theta}k_{n}{\int}_{\phi-\theta}^{\phi+\theta}\frac1{k_{n}} \leq D(\phi,\theta):=\frac{c_{0}}{\sin(\theta)^{2}}{\int}_{\phi-\theta}^{\phi+\theta}k_{n}{\int}_{\phi-\theta}^{\phi+\theta}\frac1{k_{n}}. $$
(A.35)

The function (ϕ,𝜃)↦D(ϕ,𝜃) is continuous on \([-\pi ,\pi ]\times ]0,\frac \pi 6]\). Let us now show that D is bounded in a neighborhood of 𝜃 = 0 +. Note that

$$ \begin{array}{@{}rcl@{}} D(\phi,\theta) \leq\frac{4c_{0}\theta^{2}}{\sin(\theta)^{2}} \frac{\sup_{[\phi-\theta,\phi+\theta]}k_{n}}{\inf_{[\phi-\theta,\phi+\theta]}k_{n}} \leq 8c_{0}\frac{\sup_{[\phi-\theta,\phi+\theta]}k_{n}}{\inf_{[\phi-\theta,\phi+\theta]}k_{n}} \end{array} $$
(A.36)

since \(\frac {\theta ^{2}}{\sin \limits (\theta )^{2}}<2\) for all \(\theta \in [0,\frac {\pi }6]\).

Fix \(\theta _{0}\in ]0,\frac {\pi }{6}[\). We have \(\sup _{(\phi ,\theta )\in [-\pi ,\pi ]\times [\theta _{0},\frac \pi 6]} D(\phi ,\theta )<\infty \). From Lemma A.10, we get that there are 0 < c1 < C1 and 0 < c2 < C2 such that

$$ \begin{array}{@{}rcl@{}} c_{1}&\leq &k_{n}(\phi)\leq C_{1}, \text{ for }\phi\in[-\pi,\pi]\setminus [-\theta_{0},\theta_{0}] \end{array} $$
(A.37)
$$ \begin{array}{@{}rcl@{}} c_{2}\sqrt{|\log(|\phi[)|}&\leq &k_{n}(\phi)=k(\phi)\leq C_{2}\sqrt{|\log(|\phi|)|}, \text{ for }\phi\in[-3\theta_{0},3\theta_{0}]. \end{array} $$
(A.38)

Let 𝜃 < 𝜃0. Without loss of generality, we assume that ϕ ∈ [0,π]. If ϕ𝜃𝜃0, we get [ϕ𝜃,ϕ + 𝜃] ⊂ [𝜃0,π + 𝜃0] and we get, using Eq. A.37, \(D(\phi ,\theta )\leq \frac {4C_{1}}{c_{1}}\).

If ϕ𝜃 < 𝜃0, then [ϕ𝜃,ϕ + 𝜃] ⊂ [−𝜃0, 3𝜃0], and we consider again two subcases. First subcase: ϕ ≥ 2𝜃, let u := ϕ + 𝜃 ≥ 3𝜃,

$$ \begin{array}{@{}rcl@{}} D(\phi,\theta) &\leq& \frac{8c_{0}C_{2}}{c_{2}} \left( \frac{\log(u-2\theta)}{\log(u)}\right)^{\frac12} \leq \frac{8c_{0}C_{2}}{c_{2}} \left( 1+ \frac{\log(1-\frac{2\theta}{u})}{\log(u)}\right)^{\frac12} \end{array} $$
(A.39)
$$ \begin{array}{@{}rcl@{}} &\leq& \frac{8c_{0}C_{2}}{c_{2}} \left( 1+ \frac{\log(1-\frac{2}{3})}{\log(u)}\right)^{\frac12} \leq \frac{8c_{0}C_{2}}{c_{2}} \left( 1+ \frac{\log(\frac1{3})}{\log(3\theta_{0})}\right)^{\frac12}. \end{array} $$
(A.40)

In the second subcase we have ϕ < 2𝜃. We start back from Eq. A.35:

$$ \begin{array}{@{}rcl@{}} D(\phi,\theta)\leq\frac{c_{0}}{\sin(\theta)^{2}}{\int}_{0}^{\phi+\theta}k_{n}{\int}_{0}^{\phi+\theta}\frac1{k_{n}} \leq C_{2}(\theta):=\frac{c_{0}}{\sin(\theta)^{2}}{\int}_{0}^{3\theta}k_{n}{\int}_{0}^{3\theta}\frac1{k_{n}}. \end{array} $$
(A.41)

C2 is a continuous function on ]0,𝜃0] and as 𝜃 → 0+, we get (using Lemma A.10

$$ \begin{array}{@{}rcl@{}} {\int}_{0}^{3\theta}k_{n}(\phi)d\phi \sim\frac{3\theta}{2c}\sqrt{-\log(3\theta)} \end{array} $$
(A.42)
$$ \begin{array}{@{}rcl@{}} {\int}_{0}^{3\theta}k_{n}(\phi)^{-1}d\phi \sim 6c\theta\frac1{\sqrt{-\log(3\theta)}}. \end{array} $$
(A.43)

Thus \(\lim _{\theta \to 0^{+}} C_{2}(\theta )=18.\) Finally, this gives us the result. □

With Lemma A.11, conditions of Lemma A.1 are satisfied and condition (iii) of Lemma A.2 holds. Therefore, \(W^{1}({\Omega },\omega )={H^{1}_{0}}({\Omega },\omega )\). □

Rights and permissions

Reprints and permissions

About this article

Check for updates. Verify currency and authenticity via CrossMark

Cite this article

Barret, F., Raimond, O. An Averaging Principle for Stochastic Flows and Convergence of Non-Symmetric Dirichlet Forms. Potential Anal 56, 483–548 (2022). https://doi.org/10.1007/s11118-020-09893-x

Download citation

  • Received:

  • Accepted:

  • Published:

  • Issue Date:

  • DOI: https://doi.org/10.1007/s11118-020-09893-x

Keywords

Mathematics Subject Classification (2010)

Navigation